Sunteți pe pagina 1din 19

CONSTITUTIONAL LAW II : ARREST SEARCHES AND SEIZURES

BROWN VS SECRETARY OF EDUCATION, 347 US


483 (1955)

adopted in Plessy v. Ferguson, which applied to


transportation, has no place in the field of public
education.

WARREN, C.J.
Facts: This case is a consolidation of several different
cases from Kansas, South Carolina, Virginia, and
Delaware. Several black children (through their legal
representatives, Ps) sought admission to public
schools that required or permitted segregation based
on race. The plaintiffs alleged that segregation was
unconstitutional under the Equal Protection Clause of
the Fourteenth Amendment.
In all but one case, a three judge federal district
court cited Plessy v. Ferguson in denying relief under
the separate but equal doctrine. On appeal to the
Supreme Court, the plaintiffs contended that
segregated schools were not and could not be made
equal and that they were therefore deprived of equal
protection of the laws.
Issue: Is the race-based segregation of children into
separate but equal public schools constitutional?
Held: No. The race-based segregation of children into
separate but equal public schools violates the
Equal Protection Clause of the Fourteenth
Amendment and is unconstitutional.
Segregation of children in the public schools solely on
the basis of race denies to black children the equal
protection of the laws guaranteed by the Fourteenth
Amendment, even though the physical facilities and
other may be equal. Education in public schools is a
right which must be made available to all on equal
terms.
The question presented in these cases must be
determined not on the basis of conditions existing
when the Fourteenth Amendment was adopted, but
in the light of the role of public education in
American life today. The separate but equal doctrine

Separating black children from others solely because


of their race generates a feeling of inferiority as to
their status in the community that may affect their
hearts and minds in a way unlikely ever to be
undone. The impact of segregation is greater when it
has the sanction of law. A sense of inferiority affects
the motivation of a child to learn. Segregation with
the sanction of law tends to impede the educational
and mental development of black children and
deprives them of some of the benefits they would
receive in an integrated school system. Whatever
may have been the extent of psychological
knowledge at the time of Plessy v. Ferguson, this
finding is amply supported by modern authority and
any language to the contrary in Plessy v. Ferguson is
rejected.
TAXICAB OPERATORS OF METRO MANILA VS
BOARD OF TRANSPORATION,
G.R. No. L-59234 | 1982-09-30
MELENCIO-HERRERA, J.:
FACTS: Petitioner assailed the constitutionality of an
administrative regulation phasing out taxicabs more
than six years old on grounds that it is violative of
the constitutional rights of equal protection because
it is only enforced in Manila and directed solely
towards the taxi industry.
Respondents contend that the purpose of the
regulation is the promotion of safety and comfort of
the riding public from the dangers posed by old and
dilapidated taxis.
ISSUE: Whether or not an administrative regulation
phasing out taxicabs more than six years old is a
valid exercise of police power.

HELD: No, the State in the exercise of its police


power, can prescribe regulations to promote the
safety and general welfare of the people. In addition,
there is no infringement of the equal protection
clause because it is common knowledge that
taxicabs in Manila are subjected to heavier traffic
pressure and more constant use, creating a
substantial distinction from taxicabs of other places.
Petitioners alleged that the Circular in question
violates their right to equal protection of the law
because the same is being enforced in Metro Manila
only and is directed solely towards the taxi industry.
At the outset it should be pointed out that
implementation outside Metro Manila is also
envisioned in Memorandum Circular No. 77-42. To
repeat the pertinent portion:
"For an orderly implementation of this Memorandum
Circular, the rules herein shall immediately be
effective in Metro Manila. Its implementation outside
Metro Manila shall be carried out only after the
project has been implemented in Metro Manila and
only after the date has been determined by the
Board."
In fact, it is the understanding of the Court that
implementation of the Circulars in Cebu City is
already being effected, with the BOT in the process
of conducting studies regarding the operation of
taxicabs in other cities.
The Board's reason for enforcing the Circular initially
in Metro Manila is that taxicabs in this city, compared
to those of other places, are subjected to heavier
traffic pressure and more constant use. Thus is of
common knowledge. Considering that traffic
conditions are not the same in every city, a
substantial distinction exists so that infringement of
the equal protection clause can hardly be
successfully claimed.
As enunciated in the preambular clauses of the
1|Pa g e

CONSTITUTIONAL LAW II : ARREST SEARCHES AND SEIZURES

challenged BOT Circular, the overriding consideration


is the safety and comfort of the riding public from the
dangers posed by old and dilapidated taxis. The
State, in the exercise of its police power, can
prescribe regulations to promote the health, morals,
peace, good order, safety and general welfare of the
people. It can prohibit all things hurtful to comfort,
safety and welfare of society. It may also regulate
property rights. In the language of Chief Justice
Enrique M. Fernando "the necessities imposed by
public welfare may justify the exercise of
governmental authority to regulate even if thereby
certain groups may plausibly assert that their
interests are disregarded".
In so far as the non-application of the assailed
Circulars to other transportation services is
concerned, it need only be recalled that the equal
protection clause does not imply that the same
treatment be accorded all and sundry. It applies to
things or persons identically or similarly situated. It
permits of classification of the object or subject of
the law provided classification is reasonable or based
on substantial distinction, which make for real
differences, and that it must apply equally to each
member of the class. What is required under the
equal protection clause is the uniform operation by
legal means so that all persons under identical or
similar circumstance would be accorded the same
treatment both in privilege conferred and the
liabilities imposed. The challenged Circulars satisfy
the foregoing criteria.
Evident then is the conclusion that the questioned
Circulars do not suffer from any constitutional
infirmity. To declare a law unconstitutional, the
infringement of constitutional right must be clear,
categorical and undeniable.
Valmonte vs. De Villa
Facts: On 20 January 1987, the National Capital
Region District Command (NCRDC) was activated
pursuant to Letter of Instruction 02/87 of the
Philippine General Headquarters, AFP, with the

mission of conducting security operations within its


area of responsibility and peripheral areas, for the
purpose of establishing an effective territorial
defense, maintaining peace and order, and providing
an atmosphere conducive to the social, economic
and political development of the National Capital
Region. As part of its duty to maintain peace and
order, the NCRDC installed checkpoints in various
parts of Valenzuela, Metro Manila. Petitioners aver
that, because of the installation of said checkpoints,
the residents of Valenzuela are worried of being
harassed and of their safety being placed at the
arbitrary, capricious and whimsical disposition of the
military manning the checkpoints, considering that
their cars and vehicles are being subjected to regular
searches and check-ups, especially at night or at
dawn, without the benefit of a search warrant and/or
court order. Their alleged fear for their safety
increased when, at dawn of 9 July 1988, Benjamin
Parpon, a supply officer of the Municipality of
Valenzuela, Bulacan, was gunned down allegedly in
cold blood by the members of the NCRDC manning
the checkpoint along McArthur Highway at Malinta,
Valenzuela, for ignoring and/or refusing to submit
himself to the checkpoint and for continuing to speed
off inspire of warning shots fired in the air.
Issue: WON the installation of checkpoints violates
the right of the people against unreasonable
searches and seizures
Held:
Petitioner's concern for their safety and
apprehension at being harassed by the military
manning the checkpoints are not sufficient grounds
to declare the checkpoints per se, illegal. No proof
has been presented before the Court to show that, in
the course of their routine checks, the military,
indeed, committed specific violations of petitioners''
rights against unlawful search and seizure of other
rights. The constitutional right against unreasonable
searches and seizures is a personal right invocable
only by those whose rights have been infringed, or
threatened to be infringed. Not all searches and
seizures are prohibited. Those which are reasonable
are not forbidden. The setting up of the questioned
checkpoints may be considered as a security
measure to enable the NCRDC to pursue its mission
of establishing effective territorial defense and

maintaining peace and order for the benefit of the


public. Checkpoints may not also be regarded as
measures to thwart plots to destabilize the govt, in
the interest of public security. Between the inherent
right of the state to protect its existence and
promote public welfare and an individuals right
against a warrantless search w/c is, however,
reasonably conducted, the former should prevail.
True, the manning of checkpoints by the military is
susceptible of abuse by the military in the same
manner that all governmental power is susceptible of
abuse. But, at the cost of occasional inconvenience,
discomfort and even irritation to the citizen, the
checkpoints during these abnormal times, when
conducted w/in reasonable limits, are part of the
price we pay for an orderly society and a peaceful
community.

GUANZON VS. DE VILLA [181 SCRA 623; G.R.


80508; 30 JAN 1990]
Facts: The 41 petitioners alleged that the "saturation
drive" or "aerial target zoning" that were conducted
in their place (Tondo Manila) were unconstitutional.
They alleged that there is no specific target house to
be search and that there is no search warrant or
warrant of arrest served. Most of the policemen are
in their civilian clothes and without nameplates or
identification cards. The residents were rudely rouse
from their sleep by banging on the walls and
windows of their houses. The residents were at the
point of high-powered guns and herded like cows.
Men were ordered to strip down to their briefs for the
police to examine their tattoo marks. The residents
complained that they're homes were ransacked,
tossing their belongings and destroying their
valuables. Some of their money and valuables had
disappeared after the operation. The residents also
reported incidents of maulings, spot-beatings and
maltreatment. Those who were detained also
suffered mental and physical torture to extract
confessions and tactical informations. The
respondents said that such accusations were all lies.
Respondents contends that the Constitution grants to
government the power to seek and cripple
subversive movements for the maintenance of peace
in the state. The aerial target zoning were intended
to flush out subversives and criminal elements
coddled by the communities were the said drives
2|Pa g e

CONSTITUTIONAL LAW II : ARREST SEARCHES AND SEIZURES

were conducted. They said that they have


intelligently and carefully planned months ahead for
the actual operation and that local and foreign media
joined the operation to witness and record such
event.
Issue: Whether or Not the saturation drive
committed consisted of violation of human rights.
Held: It is not the police action per se which should
be prohibited rather it is the procedure used or the
methods which "offend even hardened
sensibilities" .Based on the facts stated by the
parties, it appears to have been no impediment to
securing search warrants or warrants of arrest before
any houses were searched or individuals roused from
sleep were arrested. There is no showing that the
objectives sought to be attained by the "aerial
zoning" could not be achieved even as th rights of
the squatters and low income families are fully
protected. However, the remedy should not be
brought by a tazpaer suit where not one victim
complaints and not one violator is properly charged.
In the circumstances of this taxpayers' suit, there is
no erring soldier or policeman whom the court can
order prosecuted. In the absence of clear facts no
permanent relief can be given.
In the meantime where there is showing that some
abuses were committed, the court temporary
restraint the alleged violations which are shocking to
the senses. Petition is remanded to the RTC of
Manila.

AURELIO S. ALVERO vs ARSENIO P. DIZON, et


al.,

G.R. No. L-342

May 4, 1946

FACTS: The petitioner has been accused of treason;


that at the hearing on his petition for bail, the
prosecution presented, as part of its evidence,
certain documents which had been allegedly seized

by soldiers of the United States Army, accompanied


by Filipino Guerrillas in the petitioners house. The
Petitioner further contends that the seized
documents should be returned as it obtained by
means of force and intimidation or through coercion,
those are not his personal papers but part of the files
of the New Leaders Association, which was proven to
be an organization created for the purpose of
collaborating
with
the
enemy.
Lastly,
the
presentation of the seized documents in the trial is
tantamount to compelling him to testify against
himself, in violation of his constitutional rights.

The Petitioner consented to the presentation of the


seized documents, as part of the evidence for the
prosecution, at the hearing in his petition for bail and
at the trial of the case on the merits, without having
insisted that the question of the alleged illegality of
the search and seizure of said papers and documents
should first have been directly litigated and
established by a motion.

ISSUES: Whether or not the seized documents are


legal?

Not violated by the use of evidence of articles


obtained by an unconstitutional search and seizure.
Thus, the petitioner is estopped from questioning
their admission.

COMPULSORY SELF-INCRIMINATION

Whether or not the documents seized should be


admitted as evidence in the trial court?
PURPOSE: (Adam vs New York)
HELD. No. The petition for Certiorari with Injunction
is absolutely no merit.

RATIONALE:The right of the officer and men of the


United States Army to arrest the petitioner as a
collaborationist suspect, and to seize his personal
papers is unquestionable. Also, proclamation of
General Douglas McArthur, as Commander in Chief of
the United States of Army, declaring his purpose to
remove certain citizens of the Philippines, who had
voluntarily given aid and comfort to the enemy, in
violation of the allegiance.

EXCEPTION: Important exception to the necessity


for a Search Warrant is the right of search and
seizure as an incident to a lawful arrest. A lawful
arrest may be made either while a crime is being
committed or after its commission. The right to
search includes in both instances that of searching
the person of him who is arrested, in order to find
and seize things arrested with the crime as its fruits
as the means by which it was committed.

The purpose of the constitutional provisions against


unlawful searched and seizures is to prevent
violations of private security in person and property,
and unlawful invasions of the sanctity of the home,
by officers of the law acting under legislative and
judicial sanction, and to give remedy against such
usurpations when attempted.

PEOPLE OF THE PHILIPPINES vs. ANDRE MARTI


(193 SCRA 57)
Facts: On August 14, 1987, the appellant and his
common-law wife, Shirley Reyes went to Manila
Packaging and Export Forwarders to send packages
to Zurich, Switzerland. It was received by Anita Reyes
and ask if she could inspect the packages. Shirley
refused and eventually convinced Anita to seal the
package making it ready for shipment. Before being
sent out for delivery, Job Reyes, husband of Anita and
proprietor of the courier company, conducted an
inspection of the package as part of standard
operating procedures. Upon opening the package, he
noticed a suspicious odor which made him took
sample of the substance he found inside. He reported
this to the NBI and invited agents to his office to
3|Pa g e

CONSTITUTIONAL LAW II : ARREST SEARCHES AND SEIZURES

inspect the package. In the presence of the NBI


agents, Job Reyes opened the suspicious package
and found dried-marijuana leaves inside. A case was
filed against Andre Marti in violation of R.A. 6425 and
was found guilty by the court a quo. Andre filed an
appeal in the Supreme Court claiming that his
constitutional right of privacy was violated and that
the evidence acquired from his package was
inadmissible as evidence against him.
Issue: Can the Constitutional Right of Privacy be
enforced against private individuals?
Ruling: The Supreme Court held based on the
speech of Commissioner Bernas that the Bill of Rights
governs the relationship between the individual and
the state.
The constitutional proscription against unlawful
searches and seizures therefore applies as a restraint
directed only against the government and its
agencies tasked with the enforcement of the law. It is
not meant to be invoked against acts of private
individuals. It will be recalled that Mr Job Reyes was
the one who opened the box in the presence of the
NBI agents in his place of business. The mere
presence of the NBI agents did not convert the
reasonable search effected by Mr. Reyes into a
warrantless search and siezure proscribed by the
constitution. Merely to observe and look at that
which is in plain sight is not a search.
The judgement of conviction finding appeallant guilty
beyond reasonable doubt of the crime charged was
AFFIRMED.
[G.R. No.L-32409. February 27, 1971.]
BACHE & CO. (PHIL.), INC. and FREDERICK E.
SEGGERMAN, Petitioners, v. HON. JUDGE
VIVENCIO M. RUIZ, MISAEL P. VERA, in his
capacity as Commissioner of Internal Revenue,
ARTURO LOGRONIO, RODOLFO DE LEON,
GAVINO VELASQUEZ, MIMIR DELLOSA, NICANOR
ALCORDO, JOHN DOE, JOHN DOE, JOHN DOE,
and JOHN DOE, Respondents.
San Juan, Africa, Gonzales & San Agustin, for
Petitioners.
Solicitor General Felix Q. Antonio, Assistant
Solicitor General Crispin V .Bautista, Solicitor

Pedro A. Ramirez and Special Attorney Jaime


M. Maza for Respondents.
FACTS: Respondent Misael P. Vera, Commissioner of
Internal Revenue, wrote a letter addressed to
respondent Judge Vivencio M. Ruiz requesting the
issuance of a search warrant against petitioners for
violation of Section 46(a) of the National Internal
Revenue Code, in relation to all other pertinent
provisions thereof, particularly Sections 53, 72, 73,
208 and 209, and authorizing Revenue Examiner
Rodolfo de Leon, one of herein respondents, to make
and file the application for search warrant which was
attached to the letter.
In the afternoon of the following day, February 25,
1970, respondent De Leon and his witness,
respondent Arturo Logronio, went to the Court of First
Instance of Rizal. They brought with them the
following papers: respondent Veras aforesaid letterrequest; an application for search warrant already
filled up but still unsigned by respondent De Leon; an
affidavit of respondent Logronio subscribed before
respondent De Leon; a deposition in printed form of
respondent Logronio already accomplished and
signed by him but not yet subscribed; and a search
warrant already accomplished but still unsigned by
respondent Judge.
At that time respondent Judge was hearing a certain
case; so, by means of a note, he instructed his
Deputy Clerk of Court to take the depositions of
respondents De Leon and Logronio. After the session
had adjourned, respondent Judge was informed that
the depositions had already been taken. The
stenographer, upon request of respondent Judge,
read to him her stenographic notes; and thereafter,
respondent Judge asked respondent Logronio to take
the oath and warned him that if his deposition was
found to be false and without legal basis, he could be
charged for perjury. Respondent Judge signed
respondent de Leons application for search warrant
and respondent Logronios deposition, Search
Warrant No. 2-M-70 was then sign by respondent
Judge and accordingly issued.
Three days later, or on February 28, 1970, which was
a Saturday, the BIR agents served the search warrant
to petitioners at the offices of petitioner corporation
on Ayala Avenue, Makati, Rizal. Petitioners lawyers
protested the search on the ground that no formal
complaint or transcript of testimony was attached to
the warrant. The agents nevertheless proceeded with
their search which yielded six boxes of documents.

On March 3, 1970, petitioners filed a petition with the


Court of First Instance of Rizal praying that the
search warrant be quashed, dissolved or recalled,
that preliminary prohibitory and mandatory writs of
injunction be issued, that the search warrant be
declared null and void, and that the respondents be
ordered to pay petitioners, jointly and severally,
damages and attorneys fees. On March 18, 1970,
the respondents, thru the Solicitor General, filed an
answer to the petition. After hearing, the court,
presided over by respondent Judge, issued on July 29,
1970, an order dismissing the petition for dissolution
of the search warrant. In the meantime, or on April
16, 1970, the Bureau of Internal Revenue made tax
assessments on petitioner corporation in the total
sum of P2,594,729.97, partly, if not entirely, based
on the documents thus seized.

Issue: WON the search warrant is valid

Held: No. The elements of valid search warrant are


not present in this case, the requisites are: 1. It must
be issued upon probable cause;
2. The probable cause must be determined
personally by the judge himself;
3. The determination of the existence of probable
cause must be made after examination by the judge
of the complainant and the witnesses he may
produce; and
4. The warrant must particularly describe the place
to be searched, and the persons or things to be
seized.

It the case at bar, no personal examination at all


was conducted by respondent Judge of the
complainant (respondent De Leon) and his witness
(respondent Logronio). While it is true that the
complainants application for search warrant and the
witness printed-form deposition were subscribed and
sworn to before respondent Judge, the latter did not
ask either of the two any question the answer to
which could possibly be the basis for determining
whether or not there was probable cause against
4|Pa g e

CONSTITUTIONAL LAW II : ARREST SEARCHES AND SEIZURES

herein petitioners. Indeed, the participants seem to


have attached so little significance to the matter that
notes of the proceedings before respondent Judge
were not even taken. At this juncture it may be well
to recall the salient facts. The transcript of
stenographic notes (pp. 61-76, April 1, 1970, Annex J2 of the Petition) taken at the hearing of this case in
the court below shows that per instruction of
respondent Judge, Mr.Eleodoro V. Gonzales, Special
Deputy Clerk of Court, took the depositions of the
complainant and his witness, and that stenographic
notes thereof were taken by Mrs. Gaspar. At that time
respondent Judge was at the sala hearing a case.
After respondent Judge was through with the hearing,
Deputy Clerk Gonzales, stenographer Gaspar,
complainant De Leon and witness Logronio went to
respondent Judges chamber and informed the Judge
that they had finished the depositions. Respondent
Judge then requested the stenographer to read to
him her stenographic notes. Special Deputy Clerk
Gonzales testified it.

The search warrant in question was issued for at


least four distinct offenses under the Tax Code. The
first is the violation of Sec. 46(a), Sec. 72 and Sec. 73
(the filing of income tax returns), which are
interrelated. The second is the violation of Sec. 53
(withholding of income taxes at source). The third is
the violation of Sec. 208 (unlawful pursuit of business
or occupation); and the fourth is the violation of Sec.
209 (failure to make a return of receipts, sales,
business or gross value of output actually removed or
to pay the tax due thereon). Even in their
classification the six above-mentioned provisions are
embraced in two different titles: Secs. 46(a), 53, 72
and 73 are under Title II (Income Tax); while Secs.
208 and 209 are under Title V (Privilege Tax on
Business and Occupation).

Topic: Who may invoke the Right?


The search warrant does not particularly describe the
things to be seized.
The documents, papers and effects sought to be
seized are described in Search Warrant No. 2-M-70 in
this manner:jgc:chanrobles.com.ph
"Unregistered and private books of accounts
(ledgers, journals, columnars, receipts and
disbursements books, customers ledgers); receipts
for payments received; certificates of stocks and
securities; contracts, promissory notes and deeds of
sale; telex and coded messages; business
communications, accounting and business records;
checks and check stubs; records of bank deposits
and withdrawals; and records of foreign remittances,
covering the years 1966 to 1970."cralaw virtua1aw
library
The description does not meet the requirement in Art
III, Sec. 1, of the Constitution, and of Sec. 3, Rule 126
of the Revised Rules of Court, that the warrant should
particularly describe the things to be seized.

The search warrant was issued for more than one


specific offense.

The Supreme Court made an explanation in this


case regarding this right, as qouted:

It is next contended by respondents that a


corporation is not entitled to protection against
unreasonable search and seizures. Again, we find no
merit in the contention.
"Although, for the reasons above stated, we are of
the opinion that an officer of a corporation which is
charged with a violation of a statute of the state of
its creation, or of an act of Congress passed in the
exercise of its constitutional powers, cannot refuse to
produce the books and papers of such corporation,
we do not wish to be understood as holding
that a corporation is not entitled to immunity,
under the 4th Amendment, against
unreasonable searches and seizures. A
corporation is, after all, but an association of
individuals under an assumed name and with a
distinct legal entity. In organizing itself as a
collective body it waives no constitutional
immunities appropriate to such body. Its
property cannot be taken without

compensation. It can only be proceeded


against by due process of law, and is
protected, under the 14th Amendment, against
unlawful discrimination . . ." (Hale v. Henkel,
201 U.S. 43, 50 L. ed. 652.)

Republic of the Philippines


SUPREME COURT
Manila
EN BANC
G.R. No. L-19550 June 19, 1967
HARRY S. STONEHILL, ROBERT P. BROOKS,
JOHN J. BROOKS and KARL BECK, petitioners,
vs.
HON. JOSE W. DIOKNO, in his capacity as
SECRETARY OF JUSTICE; JOSE LUKBAN, in his
capacity as Acting Director, National Bureau of
Investigation; SPECIAL PROSECUTORS PEDRO
D. CENZON, EFREN I. PLANA and MANUEL
VILLAREAL, JR. and ASST. FISCAL MANASES G.
REYES; JUDGE AMADO ROAN, Municipal Court
of Manila; JUDGE ROMAN CANSINO, Municipal
Court of Manila; JUDGE HERMOGENES CALUAG,
Court of First Instance of Rizal-Quezon City
Branch, and JUDGE DAMIAN JIMENEZ, Municipal
Court of Quezon City, respondents.
Paredes, Poblador, Cruz and Nazareno and Meer,
Meer and Meer and Juan T. David for petitioners.
Office of the Solicitor General Arturo A. Alafriz,
Assistant Solicitor General Pacifico P. de Castro,
Assistant Solicitor General Frine C. Zaballero,
Solicitor Camilo D. Quiason and Solicitor C. Padua for
respondents.
CONCEPCION, C.J.:

5|Pa g e

CONSTITUTIONAL LAW II : ARREST SEARCHES AND SEIZURES

FACTS: Upon application of the officers of the


government named on the margin hereinafter
referred to as Respondents-Prosecutors several
judges2 hereinafter referred to as RespondentsJudges issued, on different dates, a total of 42
search warrants against petitioners herein 4 and/or
the corporations of which they were officers, directed
to the any peace officer, to search the persons
above-named and/or the premises of their offices,
warehouses and/or residences, and to seize and take
possession of the following personal property to wit:
Books of accounts, financial records, vouchers,
correspondence,
receipts,
ledgers,
journals,
portfolios, credit journals, typewriters, and other
documents and/or papers showing all business
transactions
including
disbursements
receipts,
balance sheets and profit and loss statements and
Bobbins (cigarette wrappers).
as "the subject of the offense; stolen or embezzled
and proceeds or fruits of the offense," or "used or
intended to be used as the means of committing the
offense," which is described in the applications
adverted to above as "violation of Central Bank Laws,
Tariff and Customs Laws, Internal Revenue (Code)
and the Revised Penal Code."
Petitioner alleged that the aforementioned search
warrants are null and void, as contravening the
Constitution and the Rules of Court because, inter
alia: (1) they do not describe with particularity the
documents, books and things to be seized; (2) cash
money, not mentioned in the warrants, were actually
seized; (3) the warrants were issued to fish evidence
against the aforementioned petitioners in deportation
cases filed against them; (4) the searches and
seizures were made in an illegal manner; and (5) the
documents, papers and cash money seized were not
delivered to the courts that issued the warrants, to
be disposed of in accordance with law. aid petitioners
filed with the Supreme Court this original action for
certiorari.

Respondents contended that (1) the contested


search warrants are valid and have been issued in
accordance with law; (2) that the defects of said
warrants, if any, were cured by petitioners' consent;
and (3) that, in any event, the effects seized are
admissible in evidence against herein petitioners,
regardless of the alleged illegality of the
aforementioned searches and seizures.

Issue: WON the search warrant is valid


WON a corporation cannot invoke such right

Held:1) No. Two points must be stressed in


connection with this constitutional mandate, namely:
(1) that no warrant shall issue but upon probable
cause, to be determined by the judge in the manner
set forth in said provision; and (2) that the warrant
shall particularly describe the things to be seized.

None of these requirements has been complied with


in the contested warrants. Indeed, the same were
issued upon applications stating that the natural and
juridical person therein named had committed a
"violation of Central Ban Laws, Tariff and Customs
Laws, Internal Revenue (Code) and Revised Penal
Code." In other words, no specific offense had been
alleged in said applications. The averments thereof
with respect to the offense committed were abstract.
As a consequence, it was impossible for the judges
who issued the warrants to have found the existence
of probable cause, for the same presupposes the
introduction of competent proof that the party
against whom it is sought has performed particular
acts, or committed specific omissions, violating a
given provision of our criminal laws. As a matter of

fact, the applications involved in this case do not


allege any specific acts performed by herein
petitioners. It would be the legal heresy, of the
highest order, to convict anybody of a "violation of
Central Bank Laws, Tariff and Customs Laws, Internal
Revenue (Code) and Revised Penal Code," as
alleged in the aforementioned applications without
reference to any determinate provision of said laws
or
To uphold the validity of the warrants in question
would be to wipe out completely one of the most
fundamental rights guaranteed in our Constitution,
for it would place the sanctity of the domicile and the
privacy of communication and correspondence at the
mercy of the whims caprice or passion of peace
officers. This is precisely the evil sought to be
remedied by the constitutional provision above
quoted to outlaw the so-called general warrants. It
is not difficult to imagine what would happen, in
times of keen political strife, when the party in power
feels that the minority is likely to wrest it, even
though by legal means.

Thus, the documents, papers, and things seized


under the alleged authority of the warrants in
question may be split into two (2) major groups,
namely: (a) those found and seized in the offices of
the aforementioned corporations, and (b) those
found and seized in the residences of petitioners
herein.
2) The Supreme Court held that petitioners herein
have no cause of action to assail the legality of the
contested warrants and of the seizures made in
pursuance thereof, for the simple reason that said
corporations have their respective personalities,
separate and distinct from the personality of herein
petitioners, regardless of the amount of shares of
stock or of the interest of each of them in said
corporations, and whatever the offices they hold
therein may be. Indeed, it is well settled that the
legality of a seizure can be contested only by the
6|Pa g e

CONSTITUTIONAL LAW II : ARREST SEARCHES AND SEIZURES

party whose rights have been impaired thereby, and


that the objection to an unlawful search and seizure
is purely personal and cannot be availed of by third
parties. Consequently, petitioners herein may not
validly object to the use in evidence against them of
the documents, papers and things seized from the
offices and premises of the corporations adverted to
above, since the right to object to the admission of
said papers in evidence belongs exclusively to the
corporations, to whom the seized effects belong, and
may not be invoked by the corporate officers in
proceedings against them in their individual capacity.

With respect to the documents, papers and things


seized in the residences of petitioners herein, the
aforementioned resolution of June 29, 1962, lifted the
writ of preliminary injunction previously issued by
this Court, thereby, in effect, restraining herein
Respondents-Prosecutors from using them in
evidence against petitioners herein.
In connection with said documents, papers and
things, two (2) important questions need be settled,
namely: (1) whether the search warrants in question,
and the searches and seizures made under the
authority thereof, are valid or not, and (2) if the
answer to the preceding question is in the negative,
whether said documents, papers and things may be
used in evidence against petitioners herein.
Petitioners maintain that the aforementioned search
warrants are in the nature of general warrants and
that accordingly, the seizures effected upon the
authority there of are null and void. In this
connection, the Constitution provides:
The right of the people to be secure in their persons,
houses, papers, and effects against unreasonable
searches and seizures shall not be violated, and no
warrants shall issue but upon probable cause, to be
determined by the judge after examination under
oath or affirmation of the complainant and the

witnesses he may produce, and particularly


describing the place to be searched, and the persons
or things to be seized.

Anti-Usury board to retain the articles seized, be


declared illegal and set aside, and prays that all the
articles in question be returned to him.
Issues:

Alvarez vs. The Court of First Instance


64 Phil. 33 (GR No. L-45358)
January 29, 1937

J. Imperial
Facts:
On June 3, 1936, the chief of of the secret service of
the Anti-Usury Board presented to Judge David,
presiding judge of CFI of Tayabas, alleging that
according to reliable information, the petitioner is
keeping in his house in Infanta, Tayabas documents,
receipts, lists, chits and other papers used by him in
connection with his activities as a money lender
charging usurious rates of interest in violation of the
law.
In his oath the chief of the secret service did not
swear to the truth of his statements upon his
knowledge of the facts but the information received
by him from a reliable person. Upon this questioned
affidavit, the judge issued the search warrant,
ordering the search of the petitioners house at any
time of the day or night, the seizure of the books and
documents and the immediate delivery of such to
him (judge). With said warrant, several agents of the
Anti-Usury Board entered the petitioner's store and
residence at 7 o'clock of the night and seized and
took possession of various articles belonging to the
petitioner.
The petitioner asks that the warrant of issued by the
Court of First Instance of Tayabas, ordering the
search of his house and the seizure, at anytime of
the day or night, of certain accounting books,
documents, and papers belonging to him in his
residence situated in Infanta, Tayabas, as well as the
order of a later date, authorizing the agents of the

Whether or not there is a valid search warrant


Held:
A search warrant is an order in writing, issued in the
name of the People of the Philippine Islands, signed
by a judge or a justice of the peace, and directed to a
peace officer, commanding him to search for
personal property and bring it before the court. Of all
the rights of a citizen, few are of greater importance
or more essential to his peace and happiness than
the right of personal security, and that involves the
exemption of his private affairs, books, and papers
from the inspection and scrutiny of others. While the
power to search and seize is necessary to the public
welfare, still it must be exercised and the law
enforced without transgressing the constitutional
rights or citizen, for the enforcement of no statue is
of sufficient importance to justify indifference to
thebasis principles of government.
As the protection of the citizen and the maintenance
of his constitutional right is one of the highest duties
and privileges of the court, these constitutional
guaranties should be given a liberal construction or a
strict construction in favor of the individual, to
prevent stealthy encroachment upon, or gradual
depreciation on, the rights secured by them. Since
the proceeding is a drastic one, it is the general rule
that statutes authorizing searches and seizure or
search warrants must be strictly construed.
Unreasonable searches and seizures are a menace
against which the constitutional guarantee afford full
protection. The term "unreasonable search and
seizure" is not defined in the Constitution or in
General Orders No. 58, and it is said to have no fixed,
absolute or unchangeable meaning, although the
term has been defined in general language. All illegal
searches and seizure are unreasonable while lawful
ones are reasonable. What constitutes a reasonable
or unreasonable search or seizure in any particular
case is purely a judicial question, determinable from
a consideration of the circumstances involved,
including the purpose of the search, the presence or
absence or probable cause, the manner in which the
7|Pa g e

CONSTITUTIONAL LAW II : ARREST SEARCHES AND SEIZURES

search and seizure was made, the place or thing


searched, and the character of the articles procured.
Neither the Constitution nor General Orders. No. 58
provides that it is of imperative necessity to take the
deposition of the witnesses to be presented by the
applicant or complainant in addition to the affidavit
of the latter. The purpose of both in requiring the
presentation of depositions is nothing more than to
satisfy the committing magistrate of the existence of
probable cause. Therefore, if the affidavit of the
applicant or complainant is sufficient, the judge may
dispense with that of other witnesses. Inasmuch as
the affidavit of the agent in this case was insufficient
because his knowledge of the facts was not personal
but merely hearsay, it is the duty of the judge to
require the affidavit of one or more witnesses for the
purpose of determining the existence of probable
cause to warrant the issuance of the search warrant.
When the affidavit of the applicant of the complaint
contains sufficient facts within his personal and direct
knowledge, it is sufficient if the judge is satisfied that
there exist probable cause; when the applicant's
knowledge of the facts is mere hearsay, the affidavit
of one or more witnesses having a personal
knowledge of the fact is necessary. We conclude,
therefore, that the warrant issued is likewise illegal
because it was based only on the affidavit of the
agent who had no personal knowledge of the facts.
Section 101 of General Orders, No. 58 authorizes that
the search be made at night when it is positively
asserted in the affidavits that the property is on the
person or in the place ordered to be searched. As we
have declared the affidavits insufficient and the
warrant issued exclusively upon it illegal, our
conclusion is that the contention is equally well
founded and that the search could not legally be
made at night.
The only description of the articles given in the
affidavit presented to the judge was as follows: "that
there are being kept in said premises books,
documents, receipts, lists, chits and other papers
used by him in connection with his activities as
money-lender, charging a usurious rate of interest, in
violation of the law." Taking into consideration the
nature of the article so described, it is clear that no
other more adequate and detailed description could
have been given, particularly because it is difficult to
give a particular description of the contents thereof.
The description so made substantially complies with
the legal provisions because the officer of the law

who executed the warrant was thereby placed in a


position enabling him to identify the articles, which
he did.
At the hearing of the incidents of the case raised
before the court it clearly appeared that the books
and documents had really been seized to enable the
Anti-Usury Board to conduct an investigation and
later use all or some of the articles in question as
evidence against the petitioner in the criminal cases
that may be filed against him. The seizure of books
and documents by means of a search warrant, for the
purpose of using them as evidence in a criminal case
against the person in whose possession they were
found, is unconstitutional because it makes the
warrant unreasonable, and it is equivalent to a
violation of the constitutional provision prohibiting
the compulsion of an accused to testify against
himself. Therefore, it appearing that at least nineteen
of the documents in question were seized for the
purpose of using them as evidence against the
petitioner in the criminal proceeding or proceedings
for violation against him, we hold that the search
warrant issued is illegal and that the documents
should be returned to him.
The Anti-Usury Board insinuates in its answer that
the petitioner cannot now question the validity of the
search warrant or the proceedings had subsequent to
the issuance thereof, because he has waived his
constitutional rights in proposing a compromise
whereby he agreed to pay a fine of P200 for the
purpose of evading the criminal proceeding or
proceedings. We are of the opinion that there was no
such waiver, first, because the petitioner has
emphatically denied the offer of compromise and,
second, because if there was a compromise it
reffered but to the institution of criminal proceedings
fro violation of the Anti-Usury Law. The waiver would
have been a good defense for the respondents had
the petitioner voluntarily consented to the search
and seizure of the articles in question, but such was
not the case because the petitioner protested from
the beginning and stated his protest in writing in the
insufficient inventory furnished him by the agents.
Jose Burgos vs. Chief of Staf
G.R. No L-64261
December 26, 1984

Facts:
Two warrants were issued against petitioners for the
search on the premises of Metropolitan Mail and
We Forum newspapers and the seizure of items
alleged to have been used in subversive activities.
Petitioners prayed that a writ of preliminary
mandatory and prohibitory injunction be issued for
the return of the seized articles, and that
respondents be enjoined from using the articles thus
seized as evidence against petitioner.
Petitioners questioned the warrants for the lack of
probable cause and that the two warrants issued
indicated only one and the same address. In addition,
the items seized subject to the warrant were real
properties.
Issue:
Whether or not the two warrants were valid to justify
seizure of the items.
Held:
The defect in the indication of the same address in
the two warrants was held by the court as a
typographical error and immaterial in view of the
correct determination of the place sought to be
searched set forth in the application. The purpose
and intent to search two distinct premises was
evident in the issuance of the two warrant.
As to the issue that the items seized were real
properties, the court applied the principle in the case
of Davao Sawmill Co. v. Castillo, ruling that
machinery which is movable by nature becomes
immobilized when placed by the owner of the
tenement, property or plant, but not so when placed
by a tenant, usufructuary, or any other person
having only a temporary right, unless such person
acted as the agent of the owner. In the case at bar,
petitioners did not claim to be the owners of the land
and/or building on which the machineries were
placed. This being the case, the machineries in
question, while in fact bolted to the ground remain
movable property susceptible to seizure under a
search warrant.
8|Pa g e

CONSTITUTIONAL LAW II : ARREST SEARCHES AND SEIZURES

However, the Court declared the two warrants null


and void.
Probable cause for a search is defined as such facts
and circumstances which would lead a reasonably
discreet and prudent man to believe that an offense
has been committed and that the objects sought in
connection with the offense are in the place sought
to be searched.
The Court ruled that the affidavits submitted for the
application of the warrant did not satisfy the
requirement of probable cause, the statements of the
witnesses having been mere generalizations.
Furthermore, jurisprudence tells of the prohibition on
the issuance of general warrants. (Stanford vs. State
of Texas). The description and enumeration in the
warrant of the items to be searched and seized did
not indicate with specification the subversive nature
of the said items.

Pamplona insisted on opening the bag,


which revealed dried marijuana leaves
inside. Thereafter, Mula and Molina were
handcuffed.

Mula and Molina filed a Demurrer to


Evidence, saying that the marijuna was
illegally seized from them, therefore it is
inadmissible. The trial court denied this. The
two waived presentation of evidence, and
opted to file a joint memorandum. Later, the
trial court still found them guilty, and
sentenced them to suffer the death penalty.

Pursuant to Art. 47 of the RPC and Rule 122,


Sec. 10 of the ROC, the case is elevated to
the SC on automatic review.

The SolGen moved for the acquittal of the


two.

Statement of the Case:


-

This is for review of the decision of the RTC


findingNasario Molina alias "Bobong" and
Gregorio Mula alias "Boboy" guilty of
violation of Sec. 8 of RA 6245, or the
Dangerous Drugs Act, by possessing 946.9
grants of dried marijuana.
Molina and Mula
arraingnment.

pleaded

guilty

upon

Statement of Facts:
-

People vs Molina
19 February 2001 | Ponente: Ynares-Santiago

Overview: SPO1 Paguidopon received a tip about


drug pushers. He previously caught a glimpse of one
of them, Mula, so he was able to point to him and his
companion, Molina, to arresting officers when they
were aboard a trisikad. Upon accosting them, the
police were able to find marijuana in a bag carried by
Molina, leading to their arrest. The court however
held that they were illegally arrested because their
case dont fall under the exception of an in flagrante
delicto arrest, there being no outward indication that
could justify their arrest.

On June 1996, SPO1 Marino Paguidopon


received information about a marijuana
pusher in Davao. Paguidopon first saw the
pusher in person on July of the same year,
when his informer identified Mula as the
driver of a motorcylce who just passed by
them. Molina, on the other hand, was never
identified prior arrest.
In the morning of August 8, 1996,
Paguidopon received information that the
drug pushers will pass by at NHA, Ma-a,
Davao City that morning, so he called for
assistance from the PNP. A team composed
of SPO4 Cloribel, SPO2 Paguidopon (brother
of Marino), and SPO1 Pamplona were
dispatched to proceed to Marino's housee
where they'll wait for the drug pushers will
pass by.
Two hours later, a "trisikad" identified by
Paguidopon as carrying Molina and Mula
passed by. So, the team boarded their
vehicle, overtook the trisikad and accosted
the two.

Issue and Held:


-

Applicable Laws:
Article III, Sec. 2
Article III, Sec. 3

Rationale:
-

At that point, Mula was holding a black bag.


He handed the same to Molina. Pamplona,
introducing himself as a police officer, asked
Molina to open the bag, to which Molina
replied "Boss, if possible, we will settle this."

Was the arrest of Mula and Molina fall under


the exception of in flagrante delicto in
warrantless arrests? NO

The law mandates that searches be carried


out with a search warrant upon the
existence of probable cause. Likewise, the
law protects against unreasonable searches
and seizures and holds evidence taken from
such incidents as inadmissible as evidence.
9|Pa g e

CONSTITUTIONAL LAW II : ARREST SEARCHES AND SEIZURES

There are exceptions to this, the first being


seizure conducted incidental to a lawful
arrest.

For this, there should be a lawful arrest first,


before a search can be made. It doesn't
work the other way around.

committed, is committing or is about to


commit the offense.
o

Likewise, as a rule, an arrest is legitimate if


it's with a valid warrant of arrest. However,
a police officer may conduct warrantless
arrests:
(a) In flagrante delicto - when, in his
presence, the person to be arrested has
committed, is actually committing, or is
attempting to commit an offense
(b) Arrest efected in hot pursuit - when
an offense has just been committed and
he has probable cause to believe based
on personal knowledge of facts or
circumstances that the person to be
arrested has committed it.
(c) Arrest of escaped prisoners - when
the person to be arrested is a prisoner
who
has
escaped
from
penal
establishment or a place where he is
serving final judgment or is temporarily
confined while his case is pending, or
has escaped while being transferred
from one confinement to another.

People vs Chua Ho San: The arresting


officer must have personal knowledge
that the person he is arresting has

People vsMengote: eyes darting from


side to side while holding one's
abdomen, is not indicative of probable
cause to suspect the accused of
committing the offense.

People vsEncinada: riding a motorela


while holding two plastic baby chairs is
also not indicative of probable cause

Malacatvs CA: "standing on the corner


with his eyes moving very fast and
looking at every person that come
nearer to them" is not an act
evidencing
that
the
accused
is
attempting to commit a crime. There
was no ground to even believe that the
accused was armed with a weapon
since even a telltale bulge at the front
waistline of the accused wouldn't be
visible from the officers' view.

To constitute in flagrante delcito arrests,


there are two requisites:
1.

In this case, the trial court found that the


warrantless arrest and seizure were valid
apparently because they were caught in
flagrante delicto in possession of the prohibited
drugs.
But the question is: does the
present
case
aptly
fall
within
the
exceptions to the warrant requirement? In
in flagrante delicto arrests, it is settled that
"reliable
information"
alone
is
not
sufficient to constitute probable cause that
would justify in flagrante delicto arrests.

People vsAminnudin: The accused was


just disembarking the vessel. He only
became suspect when the informer
pointed him to the officials.

The person to be arrested must execute


an overt act indicating that he has just
committed, is actually committing, or is
attempting to commit a crime

"Boss, if possible we will settle this", which


allegedly arouse the suspicion of the officers, are
not constitutive of probable cause. Were it not
for Paguidopon, the accused won't even be
identified nor found suspicious.
-

As to Mula and Molina's identity, Paguidopon


said that he conducted a surveillance of Mula,
and caught a glimpse of him on the road.
However, he doesn't even know his name or
address. As to Molina, he admits not seeing him
before the arrest. With this, Pamplona's claim
that he knew of the name of the accused is
baseless.

This case is different from People vsEncinada,


because there the officers knew the identity of
the person to be arrested. Nevertheless the
arrest was still held illegal since the accused did
not show any suspicious behavior.

It can't be said that the accused waived their


right against unreasonable searches and seizure
just because they had an implied acquiescence
to the search. This was mere passive conformity
given the circumstance.

Judgment: Mula and Molina are acquitted. Both the


arrest and the seizure are illegal.

People vs ROBERTO SALANGUIT y KO


FACTS:

2.

Such overt act is done in the presence


or within the view of the arresting
officer.

In the present case, Mula and Molina


manifested no outward indication to justify
their arrest. Holding a bag, Molina saying

A search warrant was shown to the accusedappellant and the police operatives started searching
the house. They found heat-sealed transparent
plastic bags containing a white crystalline substance,
a paper clip box also containing a white crystalline
substance, and two bricks of dried leaves which
appeared to be marijuana. A receipt of the items
seized was prepared, but the accused-appellant
refused to sign it. Charges against Roberto Salanguit
10 | P a g e

CONSTITUTIONAL LAW II : ARREST SEARCHES AND SEIZURES

y Ko for violations of Republic Act (RA) 6425, i.e. for


possession of shabu and marijuana, (Criminal Cases
Q-95-64357 and Q-95-64358, respectively) were
filed, and after hearing, the trial court convicted him
in Criminal Cases Q-95-64357 and Q-95-64358 for
violation of Section 16 and 8, respectively.
The accused-appellant contended that the evidence
against him was inadmissible because the warrant
used in obtaining it was invalid.
ISSUES:
Whether the warrant was invalid for failure of
providing evidence to support the seizure of drug
paraphernalia, and whether the marijuana may be
included as evidence in light of the plain view
doctrine.
HELD:
Yes. The warrant authorized the seizure of
undetermined quantity of shabu and drug
paraphernalia. Evidence was presented showing
probable
cause
of
the
existence
of
methamphetamine hydrochloride or shabu. The fact
that there was no probable cause to support the
application for the seizure of drug paraphernalia does
not warrant the conclusion that the search warrant is
void. This fact would be material only if drug
paraphernalia was in fact seized by the police. The
fact is that none was taken by virtue of the search
warrant issued. If at all, therefore, the search warrant
is void only insofar as it authorized the seizure of
drug paraphernalia, but it is valid as to the seizure of
methamphetamine hydrochloride as to which
evidence was presented showing probable cause as
to its existence. In sum, with respect to the seizure of
shabu from Salanguits residence, Search Warrant
160 was properly issued, such warrant being founded
on probable cause personally determined by the
judge under oath or affirmation of the deposing
witness and particularly describing the place to be
searched and the things to be seized. With respect
to, and in light of the plain view doctrine, the police
failed to allege the time when the marijuana was
found, i.e., whether prior to, or contemporaneous
with, the shabu subject of the warrant, or whether it
was recovered on Salanguits person or in an area
within his immediate control. Its recovery, therefore,
presumably during the search conducted after the
shabu had been recovered from the cabinet, as

attested to by SPO1 Badua in his deposition, was


invalid. Thus, the Court affirmed the decision as to
Criminal Case Q-95-64357 only.
Microsoft corp. v. Maxicorp
G.R No. 140946
438 SCRA 224 Mercantile Law Intellectual
Property Law on Copyright Probable Cause
in Issuing Search Warrant
FACTS:In 1996, DominadorSamiano, Jr., an agent of
the National Bureau of Investigation (NBI) conducted
a surveillance against Maxicorp, Inc. He observed
that
MicrosoftSoftwares (Windows
Operating
Systems) were being produced and packaged within
the premises of Maxicorp. Samiano, together with a
civilian witness (John Benedict Sacriz) then bought a
computer unit from Maxicorp. The unit was preinstalled with a pirated copy of Windows. For their
purchase, they were issued a receipt, however, the
receipt was in the name of a certain Joel Diaz.
Subsequently, Samiano applied for a search warrant
before the RTC. He brought with him Sacriz as
witness. He also brought the computer unit they
bought as evidence as well as the receipt. He even
added an additional witness (FelixbertoPante), a
computer technician, who showed the judge that
the software in the computer unit bought by Samiano
from Maxicorp was pirated. The
RTC judge,
convinced that there is a probable cause for a case of
copyright infringement
and unfair competition
committed by Maxicorp, issued the corresponding
warrant. Maxicorp assailed the legality of the warrant
before the Court of Appeals. The Court of Appeals
ruled in favor of Maxicorpand in its decision it
highlighted the fact that the receipt issued was not in
Samianos or Sacriz name hence the proceeding in
the trial court was infirm from the onset.
ISSUE: Whether or not the Court of Appeals is
correct.
HELD: No. The testimonies of the two witnesses,
coupled with the object and documentary evidence
they presented, are sufficient to establish the
existence of probable cause. From what they have
witnessed, there is reason to believe that Maxicorp
engaged in copyright infringement and unfair
competition to the prejudice of Microsoft. Both NBI
Agent Samiano and Sacriz were clear and insistent

that the counterfeit software were not only displayed


and sold within Maxicorps premises, they were also
produced, packaged and in some cases, installed
there.
The fact that the receipt issued was not in Samianos
name nor was it in Sacriz name does not render the
issuance of the warrant void. No law or rule states
that probable cause requires a specific kind of
evidence. No formula or fixed rule for its
determination exists. Probable cause is determined in
the light of conditions obtaining in a given
situation.Thus, it was improper for the Court of
Appeals to reverse the RTCs findings simply because
the sales receipt evidencing NBI Agent Samianos
purchase of counterfeit goods is not in his name.

Soliven v Makasiar Nov 14, 1988 G.R. No.


82585

(Topic on Warrant Issued by RTC)


Facts:Soliven broadcasted the statement that
President Aquino hid under her bed during a coup d'
etat. The President sued for libel. Soliven claimed
that he can't be sued because the President was
immune from suit.
Issue: WON Beltran's rights were violated when the
RTC issued a warrant of arrest without personally
examining the complainant and the witnesses to
determine probable cause.
Held: No
Ratio: In satisfying himself of the existence of
probable cause to issue a warrant of arrest, the judge
isn't required to examine the complainant and the
witnesses.
He shall only personally evaluate the report
and supporting documents submitted by the fiscal
regarding the existence of probable cause and issue
a warrant of arrest on the basis thereof.
Also, if he finds no probable cause, he may disregard
the fiscal's report and required the submission
of supporting affidavits of witnesses to aid him in
11 | P a g e

CONSTITUTIONAL LAW II : ARREST SEARCHES AND SEIZURES

arriving at a conclusion as to the existence of


probable cause.
Otherwise, judges would be burdened with
preliminary investigation instead of hearing cases.

Lim vs Felix

FACTS: On March 17, 1989, at about 7:30 o'clock in


the morning, at the vicinity of the airport road of the
Masbate Domestic Airport, located at the
municipality of Masbate province of Masbate,
Congressman Moises Espinosa, Sr. and his security
escorts, namely Provincial Guards Antonio Cortes,
Gaspar Amaro, and Artemio Fuentes were attacked
and killed by a lone assassin. Dante Siblante another
security escort of Congressman Espinosa, Sr.
survived the assassination plot, although, he himself
suffered a gunshot wound. An investigation of the
incident then followed.
Thereafter, and for the purpose of preliminary
investigation, the designated investigator filed an
amended complaint with the Municipal Trial Court of
Masbate accusing Vicente Lim, Sr. et al of the crime
of multiple murder and frustrated murder in
connection with the airport incident.
After conducting the preliminary investigation, the
court issued an order concluding that a probable
cause has been established for the issuance of a
warrant of arrest of named accused..
On October 30, 1989, Fiscal Alfane filed with the
Regional Trial Court of Masbate, four (4) separate
informations of murder against the twelve (12)
accused with a recommendation of no bail.
On November 21, 1989, petitioners Vicente Lim, Sr.
and Susana Lim filed with us a verified petition for
change of venue w/c was authorized, from the RTC of
Masbate to the RTCt of Makati to avoid miscarriage of
justice. The cases were raffled to Branch 56 presided
by respondent Judge Nemesio S. Felix.
Petitioners Vicente Lim, Sr. and Susana Lim filed with

the respondent court several motions and


manifestations, among others was an order be issued
requiring the transmittal of the initial records of the
preliminary inquiry or investigation conducted by the
Municipal Judge Barsaga of Masbate for the best
enlightenment of this Honorable Court in its personal
determination of the existence of a probable cause or
prima facie evidence as well as its determination of
the existence of guilt, pursuant to the mandatory
mandate of the constitution that no warrant
shall issue unless the issuing magistrate shall
have himself been personally convinced of
such probable cause.
Respondent court issued an order denying for lack of
merit the motions and manifestations and issued
warrants of arrest against the accused including the
petitioners herein.

ISSUE : Whether or not a judge may issue a warrant


of arrest without bail by simply relying on the
prosecution's certification and recommendation that
a probable cause exists.
HELD: The questioned Order of respondent Judge
Nemesio S. Felix of Branch 56, Regional Trial Court
of Makati dated July 5, 1990 isdeclared NULL and
VOID and SET ASIDE.
RD: As held in Soliven v. Makasiar, the Judge does not
have to personally examine the complainant and his
witnesses. The Prosecutor can perform the same
functions as a commissioner for the taking of the
evidence. However, there should be
necessary documents and a report supporting the
Fiscal's bare certification.
All of these should be before the Judge.
We cannot determine beforehand howcursory or exh
austive the Judge's examination Should be.
Usually, this depends on the circumstances of each
case. The Judge has to exercise sound discretion;
after all, the personal determination is vested in the
Judge by the Constitution. However, to be sure, the
Judge must go beyond the Prosecutor's certification
and investigation report whenever necessary. As
mentioned in the facts (stated above),
the Lims presented documents of recantations of the

witnesses.
Although, the general rule is that recantations are
not given much weight in the determination of a case
and in the
granting of a new trial the respondent Judge before is
suing his own warrants of arrest should, at the very
least, have gone over the records of the preliminary
examination conducted earlier in the
lightof the evidence now presented by theconcerned
witnesses in view of the"political undertones" prevaili
ng in the cases.
In making the required personaldetermination, a Judg
e is not precludedfrom relying on the evidence ealier
gathered by responsible officers. Theextent of the reli
ance depends on thecircumstances of each case and
is subject to the Judge's sound discretion. However,
the Judge abuses that discretion when having no
evidence before him, he issues a warrant of arrest.
Indubitably, the respondent Judge (Felix) committed
a grave error when he relied solely on the
Prosecutors certification and issued the questioned
Order dated July 5,1990 without having before him
any other basis for his personal determination of the
existence of a probable cause.

Panderanga vs Drilon
REGALADO, J.: p

In this special civil action for mandamus and


prohibition with prayer for a writ of preliminary
injunction/restraining order, petitioner seeks to enjoin
herein public respondents from including the former
as an accused in Criminal Case No. 86-39 for multiple
murder, through a second amended information, and
to restrain them from prosecuting him.
The records disclose that on October 16, 1986, an
information for multiple murder was filed in the
Regional Trial Court, Gingoog City, against Felipe
Galarion, Manuel Sabit, Cesar Sabit, Julito Ampo,
Eddie Torion, John Doe, Peter Doe and Richard Doe,
for the deaths on May 1, 1984 of Renato Bucag, his
wife Melchora Bucag, and their son Renato Bucag II.
Venue was, however, transferred to Cagayan de Oro
12 | P a g e

CONSTITUTIONAL LAW II : ARREST SEARCHES AND SEIZURES

City per Administrative Matter No. 87-2-244.

1990.

Only Felipe Galarion was tried and found guilty as


charged. The rest of the accused remained at large.
Felipe Galarion, however, escaped from detention
and has not been apprehended since then.

From the aforesaid resolution and order, petitioner


filed a Petition for Review 4 with the Department of
Justice. Thereafter, he submitted a Supplemental
Petition with Memorandum, 5 and then a
Supplemental Memorandum with Additional
Exculpatory/Exonerating Evidence
Annexed, 6 attaching thereto an affidavit of Roxas
dated June 20, 1990 and purporting to be a retraction
of his affidavit of March 30, 1990 wherein he
implicated herein petitioner.

In an amended information filed on October 6, 1988,


Felizardo Roxas, alias "Ely Roxas," "Fely Roxas" and
"Lolong Roxas," was included as a co-accused. Roxas
retained petitioner Paderanga as his counsel.
As counsel for Roxas, petitioner filed, among others,
an Omnibus Motion to dismiss, to Quash the Warrant
of Arrest and to Nullify the Arraignment on October
14, 1988. The trial court in an order dated January 9,
1989, denied this omnibus motion but directed the
City Prosecutor "to conduct another preliminary
investigation or reinvestigation in order to grant the
accused all the opportunity to adduce whatever
evidence he has in support of his defense."
In the course of the preliminary investigation,
through a signed affidavit, Felizardo Roxas implicated
herein petitioner in the commission of the crime
charged.
The City Prosecutor of Cagayan de Oro City inhibited
himself from further conducting the preliminary
investigation against petitioner at the instance of the
latter's counsel, per his resolution dated July 7, 1989.
In his first indorsement to the Department of Justice,
dated July 24, 1989, said city prosecutor requested
the Department of Justice to designate a state
prosecutor to continue the preliminary investigation
against herein petitioner.
In a resolution dated September 6,
1989, 1 respondent State Prosecutor Henrick F.
Gingoyon, who was designated to continue with the
conduct of the preliminary investigation against
petitioner, directed the amendment of the previously
amended information to include and implead herein
petitioner as one of the accused therein. Petitioner
moved for reconsideration, 2 contending that the
preliminary investigation was not yet completed
when said resolution was promulgated, and that he
was deprived of his right to present a corresponding
counter-affidavit and additional evidence crucial to
the determination of his alleged "linkage" to the
crime charged. The motion was, however, denied by
respondent Gingoyon in his order dated January 29,

On August 10, 1990, the Department of Justice,


through respondent Undersecretary Silvestre H. Bello
III, issued Resolution No. 648 7 dismissing the said
petition for review. His motion for reconsideration
having been likewise denied, petitioner then flied the
instant petition for mandamus and prohibition.
Issues: (1) that the preliminary investigation as to
him was not complete; and (2) that there exists
no prima facie evidence or probable cause to justify
his inclusion in the second amended information.
Ruling: Preliminary investigation is generally
inquisitorial, and it is often the only means of
discovering the persons who may be reasonably
charged with a crime, to enable the fiscal to prepare
his complaint or information. It is not a trial of the
case on the merits and has no purpose except that of
determining whether a crime has been committed
and whether there is probable cause to believe that
the accused is guilty thereof, and it does not place
the person against whom it is taken in jeopardy.
A careful analysis of the circumstances obtaining in
the present case, however, will readily show that the
same does not fall under any of the aforesaid
exceptions. Hence, the petition at bar must be
dismissed.
1. Petitioner avers that he was deprived of a full
preliminary investigation by reason of the fact that at
the time the resolution of September 6, 1989 was
issued, there were still several incidents pending
resolution such as the validity of the testimonies and
affidavits of Felizardo Roxas and Rogelio Hanopol as
bases for preliminary investigation, the polygraph
test of Roxas which he failed, and the clarificatory
questions which were supposed to be propounded by
petitioner's counsel to Roxas and Hanopol. Petitioner

likwise claims that he was deprived of the


opportunity to file his counter-affidavit to the
subpoena of April 25, 1989. These contentions are
without merit.
Firstly, it will be noted that petitioner had already
filed his counter-affidavit, pursuant to the subpoena
issued to him on April 17, 1989, wherein he
controverted the charge against him and dismissed it
as a malicious design of his political opponents and
enemies to link him to the crime. We hold that this is
sufficient compliance with the procedural
requirement of the Rules of Court, specifically Section
3(b) of Rule 112 thereof. Besides, petitioner failed to
show that the subpoena issued on April 25, 1989
involved a separate complaint charging an offense
different and distinct from that charged in the
complaint attached to the first subpoena issued to
him earlier.
Secondly, the veracity and credibility of the
witnesses and their testimonies are matters of
defense best addressed to the trial court for its
appreciation and evaluation.
Thirdly, the right of petitioner to ask clarificatory
questions is not absolute. The fiscal has the
discretion to determine whether or not he will
propound these questions to the parties or witnesses
concerned. As clearly provided for under Section
3(e), Rule 112 of the Rules of Court.:
(e) If the investigating officer believes that there are
matters to be clarified, he may set a hearing to
propound clarificatory questions to the parties or
their witnesses, during which the parties shall be
afforded an opportunity to be present but without the
right to examine or cross-examine. If the parties so
desire, they may submit questions to the to the
investigating officer which the latter may propound
to the parties or witnesses concerned.
Lastly, it has been held that "the proper forum before
which absence of preliminary investigation should be
ventilated is the Court of First Instance of a
preliminary investigation does not go to the
jurisdiction of the court but merely to the regularity
of the proceedings. It could even be waived. Indeed,
it is frequently waived. These are matters to be
inquired into by the trail court not an appellate
court." 12
13 | P a g e

CONSTITUTIONAL LAW II : ARREST SEARCHES AND SEIZURES

2. Petitioner further submits that there is no prima


facie evidence, or probable cause, or sufficient
justification to hold him to a tedious and prolonged
public trial, on the basis of the following grounds: the
questioned resolution of respondent Gingoyon is full
of factual misrepresentations or misapprehensions;
respondent's reliance on the decision of the Regional
Trial Court against Felipe Galarion suffers from
constitutional and procedural infirmities considering
that petitioner was not a party thereto, much less
was he given any opportunity to comment on or
rebut the prosecution evidence; reliance on Rogelio
Hanopol's testimony is likewise "contemptible," it
being merely hearsay in addition to the fact that
petitioner was never given the opportunity to crossexamine Hanopol at the time he testified in court;
and the affidavit of Roxas dated March 30, 1989,
which is the only evidence against petitioner, has
been rendered nugatory by his affidavit of retraction
dated June 20, 1990.
A preliminary investigation is defined as an inquiry or
proceeding for the purpose of determining whether
there is sufficient ground to engender a well founded
belief that a crime cognizable by the Regional Trial
Court has been committed and that the respondent
is probably guilty thereof, and should be held for
trial. 13 The quantum of evidence now required in
preliminary investigation is such evidence sufficient
to "engender a well founded belief as to the fact of
the commission of a crime and the respondent's
probable guilt thereof. A preliminary investigation is
not the occasion for the full and exhaustive display of
the parties' evidence; it is for the presentation of
such evidence only as may engender a wen
grounded belief that an offense has been committed
and that the accused is probably guilty thereof. 14 We
are in accord with the state prosecutor's findings in
the case at bar that there exists prima facie evidence
of petitioner's involvement in the commission of the
crime, it being sufficiently supported by the evidence
presented and the facts obtaining therein.
Likewise devoid of cogency is petitioner's argument
that the testimonies of Galarion and Hanopol are
inadmissible as to him since he was not granted the
opportunity of cross-examination.
It is a fundamental principle that the accused in a
preliminary investigation has no right to crossexamine the witnesses which the complainant may
present. Section 3, Rule 112 of the Rules of Court
expressly provides that the respondent shall only

have the right to submit a counter-affidavit, to


examine all other evidence submitted by the
complainant and, where the fiscal sets a hearing to
propound clarificatory questions to the parties or
their witnesses, to be afforded an opportunity to be
present but without the right to examine or crossexamine. Thus, even if petitioner was not given the
opportunity to cross-examine Galarion and Hanopol
at the time they were presented to testify during the
separate trial of the case against Galarion and Roxas,
he cannot assert any legal right to cross-examine
them at the preliminary investigation precisely
because such right was never available to him. The
admissibility or inadmissibility of said testimonies
should be ventilated before the trial court during the
trial proper and not in the preliminary investigation.
Furthermore, the technical rules on evidence are not
binding on the fiscal who has jurisdiction and control
over the conduct of a preliminary investigation. If by
its very nature a preliminary investigation could be
waived by the accused, we find no compelling
justification for a strict application of the evidentiary
rules. In addition, considering that under Section 8,
Rule 112 of the Rules of Court, the record of the
preliminary investigation does not form part of the
record of the case in the Regional Trial Court, then
the testimonies of Galarion and Hanopol may not be
admitted by the trial court if not presented in
evidence by the prosecuting fiscal. And, even if the
prosecution does present such testimonies, petitioner
can always object thereto and the trial court can rule
on the admissibility thereof; or the petitioner can,
during the trial, petition said court to compel the
presentation of Galarion and Hanopol for purposes of
cross-examination.

judicial discretion to determine wither or not a


criminal case should be filed in Court.

General Rule:
Injunction will not be granted to restrain a criminal
prosecution

Exception (Brocka vs Enrile):


1.
Afford adequate protection to the
constitutional rights of the accused
2.
Necessary for the orderly administration of
justice or to avoid oppression or multiplicity of
actions
3.

4.
When the acts of the officers are without or
excess of authority
5.

Double jeopardy is clearly apparent

6.
When the Court has no jurisdiction over the
offense
7.

WHEREFORE, the instant petition is hereby


DISMISSED for lack of merit.

When there is a prejudicial question

A case of persecution rather than prosecution

8.
The charges are manifestly false and
motivated by vengeance

PADERANGA vs DRILON
FACTS:
Definition of Preliminary Examination Generally
inquisitorial, often only means of discovering the
persons who may be reasonably charged with a
crime, to enable the fiscal to prepare his complaint or
information.

9.
Clearly no Prima Facie case against the
accused

The right of the accused to ask clarificatory


questions is not ABSOLUTE.

The institution of a criminal action depends upon the


sound discretion of the Fiscal. He has the quasi14 | P a g e

CONSTITUTIONAL LAW II : ARREST SEARCHES AND SEIZURES

QUANTUM OF EVIDENCE required in preliminary


investigation is such such evidence sufficient to
engender a well-founded belief as to the fact of the
omission of a crime and respondents probable guilt.

Abdula vs guiani

FACTS: On 24 June 1994, a complaint for murder,


docketed as I.S. No. 94-1361, was filed before the
Criminal Investigation Service Command, ARMM
Regional Office XII against herein petitioners and six
(6) other persons[1] in connection with the death of a
certain Abdul Dimalen, the former COMELEC
Registrar of Kabuntalan, Maguindanao.[2] The
complaint alleged that herein petitioners paid the six
other respondents the total amount of P200,000.00
for the death of Abdul Dimalen.[3]

Acting on this complaint, the Provincial Prosecutor of


Maguindanao, Salick U. Panda, in a Resolution dated
22 August 1994[4], dismissed the charges of murder
against herein petitioners and five other respondents
on a finding that there was no prima facie case for
murder against them. Prosecutor Panda, however,
recommended the filing of an information for murder
against one of the respondents, a certain Kasan
Mama. Pursuant to this Resolution, an information for
murder was thereafter filed against Kasan Mama
before the sala of respondent Judge.
In an Order dated 13 September 1994[5], respondent
Judge ordered that the case, now docketed as
Criminal Case No. 2332, be returned to the Provincial
Prosecutor for further investigation. In this Order,
respondent judge noted that although there were
eight (8) respondents in the murder case, the
information filed with the court "charged only one (1)
of the eight (8) respondents in the name of Kasan
Mama without the necessary resolution required
under Section 4, Rule 112 of the Revised Rules of
Court to show how the investigating prosecutor
arrived at such a conclusion." As such, the
respondent judge reasons, the trial court cannot
issue the warrant of arrest against Kasan Mama.

Upon the return of the records of the case to the


Office of the Provincial Prosecutor for Maguindanao, it
was assigned to 2nd Assistant Prosecutor Enok T.
Dimaraw for further investigation. In addition to the
evidence presented during the initial investigation of
the murder charge, two new affidavits of witnesses
were submitted to support the charge of murder
against herein petitioners and the other respondents
in the murder complaint. Thus, Prosecutor Dimaraw
treated the same as a refiling of the murder charge
and pursuant to law, issued subpoena to the
respondents named therein.[6] On December 6, 1994,
herein petitioners submitted and filed their joint
counter-affidavits.
After evaluation of the evidence, Prosecutor
Dimaraw, in a Resolution dated 28 December 1994,
[7]
found a prima facie case for murder against herein
petitioners and three (3) other respondents. [8] He
thus recommended the filing of charges against
herein petitioners Bai UnggieAbdula and Odin
Abdula, as principals by inducement, and against the
three (3) others, as principals by direct participation.
Likewise in this 28 December 1994 Resolution,
Provincial Prosecutor Salick U. Panda, who conducted
the earlier preliminary investigation of the murder
charge, added a notation stating that he was
inhibiting himself from the case and authorizing the
investigating prosecutor to dispose of the case
without his approval. The reasons he cited were that
the case was previously handled by him and that the
victim was the father-in-law of his son.[9]

On 2 January 1995, an information for murder dated


28 December 1994[10] was filed against the petitioner
spouses and Kasan Mama, Cuenco Usman and Jun
Mama before Branch 14 of the Regional Trial Court of
Cotabato City, then the sala of respondent judge.
This information was signed by investigating
prosecutor Enok T. Dimaraw. A notation was likewise
made on the information by Provincial Prosecutor
Panda, which explained the reason for his inhibition.

warrant, petitioners filed on 4 January 1995 an


Urgent Ex-parte Motion[13] for the setting aside of the
warrant of arrest on 4 January 1995. In this motion,
petitioners argued that the enforcement of the
warrant of arrest should be held in abeyance
considering that the information was prematurely
filed and that the petitioners intended to file a
petition for review with the Department of Justice.
A petition for review[14] was filed by the petitioners
with the Department of Justice on 11 January 1995.
[15]
Despite said filing, respondent judge did not act
upon petitioners pending Motion to Set Aside the
Warrant of Arrest.

ISSUES:
1.

Whether the Second Information for murder


filed is valid?

2.

Validity of the Warrant of Arrest issued


against petitioners.

HELD:
Petition for Certiorari and Prohibition are Granted.
RATIONALE:
In order to disqualify a Judge on the basis of
Prejudice, petitioner must prove the same by clear
and convincing evidence.
Rules of Court: No complaint or information shall be
filed or dismissed by an investigating Fiscal without
the prior written authority or approval of the
Provincial or City Fiscal or Chief of State Prosecutor. A
complaint or information can only be filed if it is
approved or authorized by the Provincial or City
Fiscal or Chief of State Prosecutor.

[11]

The following day, or on 3 January 1995, the


respondent judge issued a warrant[12] for the arrest of
petitioners. Upon learning of the issuance of the said

Soliven vs Makasiar, In satisfying himself of the


existence of probable cause, the Judge is not
required to personally examine the complainant and
his witnesses.
Ho vs People, In the case at bench, respondent
15 | P a g e

CONSTITUTIONAL LAW II : ARREST SEARCHES AND SEIZURES

admits that the issued Warrant is questionable as


there was no reason for him to doubt the validity of
the Certification made by the Assistant Prosecutor
that a Preliminary Investigation was conducted and
that Probable Cause was found to exist as against
those charged in the information filed.
DOCTRINE, the Judge shall:
1.

Personally evaluate the report and the


supporting documents submitted byt the
fiscal regarding the existence of Probable
Cause and, on the basis , issue a warrant of
arrest

2.

If, on the basis thereof he finds no probable


cause, he may disregard the fiscals report
and require the submission of supporting
affidavits of witnesses to aid him in arriving
at a conclusion as to the existence of
Probable Cause.

Ho vs People (Inting)
1.

DETERMINATION OF PROBABLE CAUSE BY


THE PROSECUTOR:

Whether there is a reasonable ground to believe that


the accused is guilty of the offense charge and
should be held for trial is what the prosecutor passes
on.
Determination of Probable Cause by a Judge: Warrant
of Arrest
1.

The Judge should decide independently,


hence, he must have supporting evidence,
other than the Prosecutors bare report,
upon which to legally sustain his own
findings on the existence of probable cause
to issue an arrest order.

In the case at bench, respondent admits that he


issued the questioned warrant as there was "no
reason for (him) to doubt the validity of the
certification made by the Assistant Prosecutor that a
preliminary investigation was conducted and that
probable cause was found to exist as against those

charged in the information filed." The statement is an


admission that respondent relied solely and
completely on the certification made by the fiscal
that probable cause exists as against those charged
in the information and issued the challenged warrant
of arrest on the sole basis of the prosecutors findings
and recommendations. He adopted the judgment of
the prosecutor regarding the existence of probable
cause as his own.
Although the prosecutor enjoys the legal
presumption of regularity in the performance of his
official duties, which in turn gives his report the
presumption of accuracy, nothing less than the
fundamental law of the land commands the judge to
personally determine probable cause in the issuance
of warrants of arrest. A judge fails in this
constitutionally mandated duty if he relies merely on
the certification or report of the investigating officer.
To be sure, we cannot determine beforehand how
cursory or exhaustive the respondents examination
of the records should be.[42] The extent of the judges
examination depends on the exercise of his sound
discretion as the circumstances of the case require.
In the case at bench, the respondent had before him
two different informations and resolutions charging
two different sets of suspects. In the face of these
conflicting resolutions, it behooves him not to take
the certification of the investigating prosecutor at
face value. The circumstances thus require that
respondent look beyond the bare certification of the
investigating prosecutor and examine the documents
supporting the prosecutors determination of
probable cause. The inordinate haste that attended
the issuance of the warrant of arrest and
respondents own admission are circumstances that
tend to belie any pretense of the fulfillment of this
duty.
Clearly, respondent judge, by merely stating that he
had no reason to doubt the validity of the
certification made by the investigating prosecutor
has abdicated his duty under the Constitution to
determine on his own the issue of probable cause
before issuing a warrant of arrest. Consequently, the
warrant of arrest should be declared null and void.

PEOPLE VS.BENHUR MAMARIL


G.R. No. 147607. January 22, 2004

Facts: SPO2 ChitoEsmenda applied before the RTC


for a search warrant authorizing the search for
marijuana at the family residence of appellant
Benhur. During the search operation, the searching
team confiscated sachets of suspected marijuana
leaves. Police officers took pictures of the confiscated
items and prepared a receipt of the property seized
and certified that the house was properly searched
which was signed by the appellant and the barangay
officials who witnessed the search.
After the search, the police officers brought appellant
and the confiscated articles to the PNP station. After
weighing the specimens and testing the same, the
PNP Crime Laboratory issued a report finding the
specimens to be positive to the test for the presence
of marijuana. Moreover, the person who conducted
the examination on the urine sample of appellant
affirmed that it was positive for the same.
Appellant denied that he was residing at his parents
house since he has been residing at a rented house
and declared that it was his brother and the latters
family who were residing with his mother, but on said
search operation, his brother and family were out. He
testified that he was at his parents house because
he visited his mother, that he saw the Receipt of
Property Seized for the first time during the trial and
admitted that the signature on the certification that
the house was properly search was his.
Issues: 1) Whether or not the trial court erred in
issuing a search warrant.
2) Whether or not the accused-appellant waived his
right to question the legality of the search.
3) Whether or not evidence seized pursuant to an
illegal search be used as evidence against the
accused.
Held: 1) The issuance of a search warrant is justified
only upon a finding of probable cause. Probable
cause for a search has been defined as such facts
and circumstances which would lead a reasonably
discreet and prudent man to believe that an offense
has been committed and that the objects sought in
connection with the offense are in the place sought
to be searched. In determining the existence of
probable cause, it is required that: 1) The judge must
examine the complaint and his witnesses personally;
2) the examination must be under oath; 3) the
examination must be reduced in writing in the form
of searching questions and answers. The prosecution
failed to prove that the judge who issued the warrant
put into writing his examination of the applicant and
16 | P a g e

CONSTITUTIONAL LAW II : ARREST SEARCHES AND SEIZURES

his witnesses on the form of searching questions and


answers before issuance of the search warrant. Mere
affidavits of the complainant and his witnesses are
not sufficient. Such written examination is necessary
in order that the judge may be able to properly
determine the existence and non-existence of
probable cause. Therefore, the search warrant is
tainted with illegality by failure of the judge to
conform with the essential requisites of taking the
examination in writing and attaching to the record,
rendering the search warrant invalid.
2) At that time the police officers presented the
search warrant, appellant could not determine if the
search warrant was issued in accordance with law. It
was only during the trial that appellant, through his
counsel, had reason to believe that the search
warrant was illegally issued. Moreover, appellant
seasonably objected on constitutional grounds to the
admissibility of the evidence seized pursuant to said
warrant during the trial, after the prosecution
formally offered its evidence. Under the
circumstances, no intent to waive his rights can
reasonably be inferred from his conduct before or
during the trial.
3) No matter how incriminating the articles taken
from the appellant may be, their seizure cannot
validate an invalid warrant. The requirement
mandated by the law that the examination of the
complaint and his witnesses must be under oath and
reduced to writing in the form of searching questions
and answers was not complied with, rendering the
search warrant invalid. Consequently, the evidence
seized pursuant to illegal search warrant cannot be
used in evidence against appellant in accordance
with Section 3 (2) Article III of the Constitution.

People vs De Los Reyes G.R. 140657


FACTS: On June 18, 1998, SPO3 Benjamin Nuguid of
the Western Police District applied for a search
warrant with the RTC of Manila, Branch 43, against
Cesar Reyes alias "Cesar Itlog." In support of his
application, Nuguid submitted his affidavit and that
of Alexis Tan, a housewife. Nuguid and Tan also
testified in support of the application. After the court
conducted examination of the said witnesses, it
issued on even date Search Warrant No. 98-905
authorizing the search of the house allegedly under
the possession and custody of one Cesar Reyes alias
"Cesar Itlog," at No. 2600 Oroquieta Street, Sta. Cruz,
Manila.

The policemen conducted a search not only of the


house at No. 2600 Oroquieta Street, Sta Cruz, Manila,
which turned out to be the house of respondent
Cesar delos Reyes, but also of the car and motorcycle
owned by De Los Reyes. They found several firearms,
38 pcs of valium, 18 transparent plastic bags with
total net weight of 886.8 grams of white crystalline
substance known as "shabu".
The respondent filed a motion to quash the
informations but was denied by the trial court
The respondent then filed a petition for certiorari in
the CA, alleging that the questions propounded by
RTC Judge Manuela F. Lorenzo on Nuguid and Tan
were leading and not searching. He also alleged that
Judge Lorenzo delegated the examination of Tan to
Nuguid, and allowed the latter to question her.
The CA rendered a Decision granting the petition and
nullifying the search warrant and held that the RTC
delegated its duty to determine probable cause to
the applicant; the application for a search warrant
was based on hearsay evidence;
The People of the Philippines filed the instant petition
for review of the decision. The petitioner also asserts
that the leading questions propounded by Judge
Lorenzo on Tan does not detract from the fact that
searching questions were also propounded on the
witnesses, and that based on the entirety of such
propounded questions and the latters answers, there
was probable cause for the issuance of a search
warrant.
ISSUE: WON the search warrant issued by the judge
is valid.
HELD: The mandate of the Judge is for him to
conduct a full and searching examination of the
complainant and the witnesses he may produce. In
the absence of a rule to the contrary, the
determination of probable cause cannot be delegated
by the Judge, in part, or in whole, regardless of the
qualifications of the person on whom reliance is
placed. It is not permissible for the Judge to share the
required determination with another.
The searching questions propounded to the applicant
and the witnesses must depend on a large extent
upon the discretion of the Judge. Although there is no

hard-and-fast rule as to how a Judge may conduct his


examination, it is axiomatic that the said
examination must be probing and exhaustive and not
merely routinary, general, peripheral or perfunctory.
The questions propounded on Nuguid by Judge
Lorenzo were not searching and probing, but merely
superficial and perfunctory.
Even a cursory reading of the transcript will show
that most of the questions propounded on Tan by the
Judge were leading questions, and that those which
were not leading were merely based on or related to
the answers earlier given to the leading questions.
By asking such leading questions, the Judge thereby
supplied the answers to her questions.
Thus, in issuing a search warrant, the Judge must
strictly comply with the requirements of the
Constitution and the statutory provisions. A liberal
construction should be given in favor of the
individual to prevent stealthy encroachment upon, or
gradual depreciation of the rights secured by the
Constitution. No presumption of regularity is to be
invoked in aid of the process when an officer
undertakes to justify it. PETITION IS DENIED.

Pasion Vda. De Garcia vs. Locsin G.R. No. L45950


FACTS: Mariano G. Almeda, an agent of the AntiUsuary Board, obtained from the justice of the peace
of Tarlac, a search warrant commanding any officer
of the law to search the person, house or store of the
petitioner at Victoria, Tarlac, for certain books, lists,
chits, receipts, documents and other papers relating
to her activities as usurer. The search warrant was
issued upon an affidavit given by the said Almeda.
On the same date, the said Mariano G. Almeda,
accompanied by a captain of the Philippine
Constabulary, went to the office of the petitioner in
Victoria, Tarlac and, after showing the search warrant
to the petitioners bookkeeper, Alfredo Salas, and,
without the presence of the petitioner who was ill
and confined at the time, proceeded with the
execution thereof
The papers and documents seized were kept for a
17 | P a g e

CONSTITUTIONAL LAW II : ARREST SEARCHES AND SEIZURES

considerable length of time by the Anti-Usury Board


and thereafter were turned over by it to the
respondent fiscal who subsequently filed six separate
criminal cases against the herein petitioner for
violation of the Anti-Usury Law.
The legality of the search warrant was challenged by
counsel for the petitioner in the six criminal cases
and the devolution of the documents demanded. The
respondent Judge denied the petitioners motion for
the reason that though the search warrant was
illegal, there was a waiver on the part of the
petitioner.
HELD: Freedom from unreasonable searches and
seizures is declared a popular right and for a search
warrant to be valid, (1) it must be issued upon
probable cause; (2) the probable cause must be
determined by the judge himself and not by
the applicant or any other person; (3) in the
determination of probable cause, the judge must
examine, under oath or affirmation, the complainant
and such witnesses as the latter may produce; and
(4) the warrant issued must particularly describe the
place to be searched and persons or things to be
seized.
In the instant case the existence of probable
cause was determined not by the judge himself
but by the applicant. All that the judge did was
to accept as true the affidavit made by agent
Almeda. He did not decide for himself. It does
not appear that he examined the applicant and his
witnesses, if any. Even accepting the description of
the properties to be seized to be sufficient and on the
assumption that the receipt issued is sufficiently
detailed within the meaning of the law, the
properties seized were not delivered to the court
which issued the warrant, as required by law.
Instead, they were turned over to the resp. provincial
fiscal & used by him in building up cases against
petitioner. Considering that at the time the warrant
was issued, there was no case pending against the
petitioner, the averment that the warrant was issued
primarily for exploration purposes is not without
basis.

People vs De Los Reyes G.R. 140657

FACTS: On June 18, 1998, SPO3 Benjamin Nuguid of


the Western Police District applied for a search
warrant with the RTC of Manila, Branch 43, against
Cesar Reyes alias "Cesar Itlog." In support of his
application, Nuguid submitted his affidavit and that
of Alexis Tan, a housewife. Nuguid and Tan also
testified in support of the application. After the court
conducted examination of the said witnesses, it
issued on even date Search Warrant No. 98-905
authorizing the search of the house allegedly under
the possession and custody of one Cesar Reyes alias
"Cesar Itlog," at No. 2600 Oroquieta Street, Sta. Cruz,
Manila.
The policemen conducted a search not only of the
house at No. 2600 Oroquieta Street, Sta Cruz, Manila,
which turned out to be the house of respondent
Cesar delos Reyes, but also of the car and motorcycle
owned by De Los Reyes. They found several firearms,
38 pcs of valium, 18 transparent plastic bags with
total net weight of 886.8 grams of white crystalline
substance known as "shabu".
The respondent filed a motion to quash
informations but was denied by the trial court

the

The respondent then filed a petition for certiorari in


the CA, alleging that the questions propounded by
RTC Judge Manuela F. Lorenzo on Nuguid and Tan
were leading and not searching. He also alleged that
Judge Lorenzo delegated the examination of Tan to
Nuguid, and allowed the latter to question her.
The CA rendered a Decision granting the petition and
nullifying the search warrant and held that the RTC
delegated its duty to determine probable cause to
the applicant; the application for a search warrant
was based on hearsay evidence;
The People of the Philippines filed the instant petition
for review of the decision. The petitioner also asserts
that the leading questions propounded by Judge
Lorenzo on Tan does not detract from the fact that
searching questions were also propounded on the
witnesses, and that based on the entirety of such
propounded questions and the latters answers, there
was probable cause for the issuance of a search
warrant.
ISSUE: WON the search warrant issued by the judge
is valid.

HELD: The mandate of the Judge is for him to


conduct a full and searching examination of the
complainant and the witnesses he may produce. In
the absence of a rule to the contrary, the
determination of probable cause cannot be delegated
by the Judge, in part, or in whole, regardless of the
qualifications of the person on whom reliance is
placed. It is not permissible for the Judge to share the
required determination with another.
The searching questions propounded to the applicant
and the witnesses must depend on a large extent
upon the discretion of the Judge. Although there is no
hard-and-fast rule as to how a Judge may conduct his
examination, it is axiomatic that the said
examination must be probing and exhaustive and not
merely routinary, general, peripheral or perfunctory.
The questions propounded on Nuguid by Judge
Lorenzo were not searching and probing, but merely
superficial and perfunctory.
Even a cursory reading of the transcript will show
that most of the questions propounded on Tan by the
Judge were leading questions, and that those which
were not leading were merely based on or related to
the answers earlier given to the leading questions.
By asking such leading questions, the Judge thereby
supplied the answers to her questions.
Thus, in issuing a search warrant, the Judge must
strictly comply with the requirements of the
Constitution and the statutory provisions. A liberal
construction should be given in favor of the
individual to prevent stealthy encroachment upon, or
gradual depreciation of the rights secured by the
Constitution. No presumption of regularity is to be
invoked in aid of the process when an officer
undertakes to justify it. PETITION IS DENIED.
Pasion Vda. De Garcia vs. Locsin G.R. No. L45950
FACTS: Mariano G. Almeda, an agent of the AntiUsuary Board, obtained from the justice of the peace
of Tarlac, a search warrant commanding any officer
of the law to search the person, house or store of the
petitioner at Victoria, Tarlac, for certain books, lists,
chits, receipts, documents and other papers relating
to her activities as usurer. The search warrant was
issued upon an affidavit given by the said Almeda.
18 | P a g e

CONSTITUTIONAL LAW II : ARREST SEARCHES AND SEIZURES

On the same date, the said Mariano G. Almeda,


accompanied by a captain of the Philippine
Constabulary, went to the office of the petitioner in
Victoria, Tarlac and, after showing the search warrant
to the petitioners bookkeeper, Alfredo Salas, and,
without the presence of the petitioner who was ill
and confined at the time, proceeded with the
execution thereof

illegal, there was a waiver on the part of the


petitioner.

The papers and documents seized were kept for a


considerable length of time by the Anti-Usury Board
and thereafter were turned over by it to the
respondent fiscal who subsequently filed six separate
criminal cases against the herein petitioner for
violation of the Anti-Usury Law.

HELD: Freedom from unreasonable searches and


seizures is declared a popular right and for a search
warrant to be valid, (1) it must be issued upon
probable cause; (2) the probable cause must be
determined by the judge himself and not by the
applicant or any other person; (3) in the
determination of probable cause, the judge must
examine, under oath or affirmation, the complainant
and such witnesses as the latter may produce; and
(4) the warrant issued must particularly describe the
place to be searched and persons or things to be
seized.

The legality of the search warrant was challenged by


counsel for the petitioner in the six criminal cases
and the devolution of the documents demanded. The
respondent Judge denied the petitioners motion for
the reason that though the search warrant was

In the instant case the existence of probable cause


was determined not by the judge himself but by the
applicant. All that the judge did was to accept as true
the affidavit made by agent Almeda. He did not
decide for himself. It does not appear that he

examined the applicant and his witnesses, if any.


Even accepting the description of the properties to
be seized to be sufficient and on the assumption that
the receipt issued is sufficiently detailed within the
meaning of the law, the properties seized were not
delivered to the court which issued the warrant, as
required by law.
Instead, they were turned over to the resp. provincial
fiscal & used by him in building up cases against
petitioner. Considering that at the time the warrant
was issued, there was no case pending against the
petitioner, the averment that the warrant was issued
primarily for exploration purposes is not without
basis.

19 | P a g e

S-ar putea să vă placă și